To make meatloaf. mix 2 pounds of beef and 0.5 of a cup of breadcrumbs. How many pounds of beef
would you mix with 4.5 cups of breadcrumbs

Answers

Answer 1

Answer:

18 pounds of beef.

Step-by-step explanation:

For every 0.5 cups of breadcrumbs there's 2 pounds of beef.

0.5 x 9 = 4.5

9 x 2 = 18 or 2 x 9 = 18

multiples of two: 2 4 6 8 10 12 14 16 18

This shows that when we multiply 2 nine times we get 18!


Related Questions

help me please i will give brainliest and excellent and thank you






math​

Answers

Answer:

D

Step-by-step explanation:

Answer:

D

Step-by-step explanation:

1) know your grams and centigrams!

     1 Grams = 100 Centigrams

2)  If you want to find the centigrams, all you gotta do is; multiply the gram to 100.

For example: This bar of chocolate has 100 grams of sugar. I wonder what is the mass in centigrams?

100 grams * 100 centigrams = 1,000 Centigrams.

3) To answer your question; multiply 347 by 100!

   347 * 100 = 34,700 centigrams.

Hope this helps!

Four coordinates A, B, C and D form a rectangle.
A = (2,8), B = (9, 2) and C'=(2, 2)
What are the coordinates of D?​

Answers

Answer: (9,8)

Thats the answer

1 of 12
Eli is following this recipe to bake bread rolls.
He uses 500 g of flour.
How much of the other ingredients does he use?
Recipe: Makes 12 bread rolls
400 g flour
6 g salt
8 g yeast
40 mL oil
360 ml water
g salt
g yeast
mL oil
mL water

Answers

Answer:

360

Step-by-step explanation:

&$5%4645$655$456&&&66

Please help ASAP if you know how. Thanks so much

Answers

i’m not so sure i’m sorry

Answer:

B) -6.61, 0.61

Step-by-step explanation:

Answer obtained by graphing the formula.

Test is due Wednesday and I need urgent help. I will give good review!

Answers

Answer:

B and D

Step-by-step explanation:

Answer:

b and d

Step-by-step explanation:

Find the area of the triangle

Answers

Answer:

[tex]\sqrt{20}\ or\ 2\sqrt{5}[/tex]

Step-by-step explanation:

Find the slope. (10,-6) and (13,-7)

Answers

Answer:

-1/13

Step-by-step explanation:

i did it

pls help quickly maath

Answers

It is A
Because it says squares to the circles
So you count the squares first then the circles
So it is 3/6 .

Answer:

It should be A

Step-by-step explanation:

Since it is asking for the squares to circles, it is not option c. B is also incorrect because the ratio is not correct, which leaves the only answer to be A.

B. Subtract the following integers
16.9 - (-5) =
17. - 12 - ( + 8) =
18. 15 - 9=
19. - 5 - (-8)=
20. - 11 - ( + 14 ) =​

Answers

Answer:

16. 9 - (-5) = 21.9

17. -12 - ( + 8) = -4

18. 15 - 9 = 6

19. -5 - (-8)= 3

20. -11 - ( + 14 ) =​ -25

Step-by-step explanation:

Subtracting a negative from another number is the same as adding the number.  Thus, 16.9 - (-5) = 16.9 + 5.

Hope it helps <3 :)

Answer:

16.9-(-5)=21.9

17.-12-(+8)= -3

18.15-9=9.15

19.-5-(-8)=22

20.-11-(+14)=-5

In the image shown line n is a transversal cutting parallel lines l and m
A- 79
B- 81
C- 83
D- 86

Answers

the answer to your question is C

Sam says: 9.05kg is equal to 9500g. Is he right or wrong? Explain your answer.

Answers

Answer:

Step-by-step explanation:

Sam is wrong because 9,05 kg =9050g which is different to 9500g

Answer:

wrong 1 kilo in a gram is 1000 so u just going to add 1 zero on the back so 905+0= 9050 grams

kate pays $203 in advance on her account at the athletic club. Each time she uses the club, $9 is deducted from the account. Find the remaining in the account after 12 visits

Answers

Answer:

95

Step-by-step explanation:

The equation to solving this would be 203 - (9 multiply 12). If you multiply 9 and 12 you get 108. Minus 108 from 203 and you get 95.

Please give me an answer with an explanation.

Answers

Answer:

40 and 10

Step-by-step explanation:

1. if the length is 'l' and height is 'h', then according to the condition 1/4*l=h.

2. if the given area is 400, it means, h*l=400;

3. if to substitute h=1/4*l into the formula of the area, then

l*l*1/4=400; ⇔ 1/4*l²=400;

4. if to calculate the value of the 'l', then l=40 (the required length).

5. if to substitute l=40 into 1/4*l=h, then h=10 (the required height).

Which of the following pairs of ratios are equivalent?
1) 38 : 133 and 16 : 56 , 2) 15 : 20 and 28 : 56

Answers

Answer:

38 : 133 and 16 : 56 are equivalent

Step-by-step explanation:

Given

1) [tex]38 : 133[/tex] and [tex]16 : 56[/tex]

2) [tex]15 : 20[/tex] and [tex]28 : 56[/tex]

Required

Determine which is equivalent

1) [tex]38 : 133[/tex] and [tex]16 : 56[/tex]

[tex]Ratio = 38 : 133[/tex]

Divide by 19

[tex]Ratio = 2:7[/tex] --- can't be further simplified

[tex]Ratio = 16 : 56[/tex]

Divide by 8

[tex]Ratio = 2:7[/tex]

Hence:

[tex]38 : 133[/tex] and [tex]16 : 56[/tex] are equivalent

2) [tex]15 : 20[/tex] and [tex]28 : 56[/tex]

[tex]Ratio = 15 : 20[/tex]

Divide by 5

[tex]Ratio = 3 : 4[/tex]

[tex]Ratio = 28 : 56[/tex]

Divide by 28

[tex]Ratio = 1:2[/tex]

Hence, both are not equivalent.

The endpoints of two radii that touch the circle become the endpoints of a section of the circle. What is the curved section between the endpoints called?
A radius
B diameter
C chord
D arc

Answers

Answer:

It is a cord

Step-by-step explanation:

i just took the quiz and got it right :)

Answer:

D. arc

Step-by-step explanation:

everyone keeps saying its "chord" but I tried that and I got it wrong...

What is the area of this parallelogram

Answers

Answer:

B. 12.25

Step-by-step explanation:

A=bxh=3.5·3.5=12.25

Answer:

17.15

Step-by-step explanation:

There are 2 ways to solve this.

One way is this:

Add 1.4 and 3.5 together to get the total length of the parallelogram. The sum would be 4.9. Then multiply this number by the height of the figure(3.5cm). So that the product of 4.9 and 3.5 is 17.15.

The second way is this:

Find the area of the two triangles and the square and then add those products together. So add:

1.4 x 3.5 ÷21.4 x 3.5 ÷23.5 x 3.5

So solve this equation and you will get:

2.45 + 2.45 + 12.25=17.15

Jeff had 20 $ he spent 1/8 on his money on lunch and 2/5 of his money at the movies. How much money does Jeff have left

Answers

Answer:

9.50$

Step-by-step explanation:

First, multiply 20 by the given fractions

20*1/8=2.5

20*2/5=8

Add the two together to get the total amount of money Jeff spent, 10.5, then subtract 10.5 from 20 to get the total amount he has left, 9.5

Answer:

$9.50

I did a test with this question and the correct answer was 9.50

Expresa el área del cuadrado interno con un polinomio,luego calcula el área
x = 4y y = 10.
Es para hoy ayudaaaaaa

Answers

Answer:

si me ayudas diciéndome de qué materia es y qué grado eres Y qué página es Con gusto te ayudo pero si no es de quinto grado página 83 y 82 del trimestre 1 de la guía tal vez no te lo pueda responder pero te envió una foto para que veas Sí sí es lo que estás buscando o no

If you have a cylinder with a volume of 282.6 ft3 and a cone with a congruent radius and height. What is the volume of the cone

Answers

The volume of a cone is 1/3 the volume of a cylinder with congruent dimensions, so:

286.6 / 3 = 95.53333... ft3

In the year 2001, a person bought a new car for $15500. For each consecutive year after that, the value of the car depreciated by 5%. How much would the car be worth in the year 2005, to the nearest hundred dollars?

Answers

Answer:

$12,000.

Step-by-step explanation:

Given that in the year 2001, a person bought a new car for $ 15500, and for each consecutive year after that, the value of the car depreciated by 5%, to determine how much would the car be worth in the year 2005, to the nearest hundred dollars, the following calculation must be performed:

100-5 = 95

15,500 x 0.95 x 0.95 x 0.95 x 0.95 x 0.95 = X

14.725 x 0.95 x 0.95 x 0.95 x 0.95 = X

13,988.75 x 0.95 x 0.95 x 0.95 = X

13,289.3125 x 0.95 x 0.95 = X

12,624.846875 x 0.95 = X

11.993.60453125 = X

Thus, to the nearest hundred dollars, the cost of the car after 5 years will be $ 12,000.

PLZ HELP!! I NEED QUICKLY!!
Choose 5 ordered pairs whose first and second coordinate are equal. Plot these points and connect them. What kind of figure do you get? In what quadrants does the figure lie?

Answers

Answer:

no worries about the other day and I had literally forgotten about the other night and was just so I could tell me how I felt and I had a flat rate that you and I had a

The figure of y = x will be a straight line with slope 1 and they ;lie in the first quadrant and the third quadrant.

What is Linear Function?

A linear function can be defined as the function whose graph is a straight line. It can also be defined as the function with one or more variables but does not have exponents to the variables or the exponent of the variable is 1.

We have to choose 5 ordered pairs first whose first and second coordinate are equal.

Let the coordinates be (-4, -4), (-2, -2), (0, 0), (2, 2) and (4, 4).

The ordered pairs are of the form (x, x), that both the coordinates have same value.

So we get a linear function y = x

The graph of a linear function will be a straight line.

Since y = x, the graph will be a straight line which tends to the positive direction and the slope of the line = 1.

That is , the angle that the line making with the X axis is 45°.

And the figure lies on first and third quadrant.

Hence the figure of y = x would be a straight line with slope 1.

To learn more about Linear Functions, click :

https://brainly.com/question/21107621

#SPJ2

(06.04 HC) The graph shows Wilson's science scores versus the number of hours spent doing science homework. A graph titled Wilsons Science Scores shows Hours of Practice in a Week on x axis and Science Scores on y axis. The x axis scale is shown from 0 to 5 at increments of 1, and the y axis scale is shown from 0 to 50 at increments of 5.The ordered pairs 0, 15 and 0.5, 18 and 1, 18 and 1.5, 25 and 2, 30 and 2.5, 35 and 3, 40 and 3.5, 43 and 4, 41 and 4.5, 45 and 5, 48 are shown on the graph. A straight line joins the ordered pairs 0, 14.9 and 5, 50. What will most likely be Wilson's approximate science score if he does science homework for 6 hours a week? (5 points) Group of answer choices 33 points 42 points 52 points 55 points

Answers

Answer:

The answer 55

Step-by-step explanation:

Because it goes up by five for they gave us the amount of point they get for 5 hours and the amount of point he had for 5 hours was 50 so we just needed to add 5 more and 50+5= 55

I don't know if I made myself clear but good luck! :)

Answer

Personally I think it's 55 as well!

Step-by-step explanation:

Good luck! :)))

The table shows values for functions f(x) and g(x)

What is the solution to f(x)=g(x) ?

Select each correct answer.




A. −3

B. −2

C. −1

D. 0

E. 1

F. 2

G. 3

Answers

Answer:

E) 1

Step-by-step explanation:

this is when both functions have the same value

Element X decays radioactively with a half life of 9 minutes. If there are 760 grams of Element X, how long, to the nearest tenth of a minute, would it take the element to decay to 19 grams?

Answers

Answer:

Step-by-step explanation:

760·0.5^(t/9) = 19

0.5^(t/9) = 19/760

2^(-t/9) = 0.025

-t/9 = log₂(0.025)

t = -9log₂(0.025) = 47.9 minutes

Answer:

47.9 minutes

Step-by-step explanation:

Does the graph below represent a function?
yes
no
sometimes
not enough information

Answers

Answer:

yes it does represent a function.  

Step-by-step explanation:

Yes

Explanation:
Passes the vertical line test

Rachelle bought x pounds of fruit for
$2 per pound and a gallon of milk for
$3.79. She spent a total of $17.29. Write
an equation that can be used to find
how many pounds of fruit she bought.

Answers

Step-by-step explanation:

step 1. 2x + 3.79 = 17.29. this is the answer.

Jenny earns $30 a day working part time at a supermarket. Write an expression to represent the amount of money she will earn in d days.

Answers

Answer: d($30)

Explanation: $30 times d which d is the number of days.

what is the solution to the equation sqrt 2x+3- sqrt x+2=2

Answers

Answer:

x= 23

Step-by-step explanation:

The correct answer on edge is 23

Find angle. Picture below!

Answers

Answer:

m<QRP=42degrees

m<Q=79degrees

m<P=59degrees

Step-by-step explanation:

Hi can someone please answer and explain all you work thank you! I’ll mark brainliest

Answers

the amount he would make if he broke none:
576 x 0.05 = 28.80

subtract what he made from this:
28.80 - 22.71 = 6.09

divide 6.09 by 1.98 to see how many glasses he broke:
6.09 / 1.98 = 3.0757

he broke 3 to 4 glasses.
Other Questions
Sally says it is possible to draw a triangle with these side length 5cm, 5cm, 12cm Is she correct? Explain how you know explain why fitness is achieve not received? If I were to visit the country of Ghana, could Ivisit the ancient ruins of the Kingdom ofGhana?NoYes (1 point)In Worcester v. Georgia, the Supreme Court ruled that Georgia did not have legauthority over the Cherokees living in the state. President Andrew Jacksonresponded to this decision by -filing another petition against the Cherokee Nationignoring the ruling and allowing Georgia to evict the Cherokee Nationordering state officials in Georgia to negotiate with the Cherokee Nationasking Congress to pay reparations to the Cherokee Nation Please help me i would appreciate it a lot Which sentence best summarizes Annes point of view regarding herself and her diary?A.) Adults would disapprove of the diary and her behavior.B.) Her thoughts arent interesting enough for others to read.C.) Everyone will want to read her diary. Answer 1. And 2.for Brainliest How does stress the fight-or-flight response affect vision explain how ? How did Japan acquire the natural resources needed for industrialization before the Great Depression Im sooo confused rn please help pleaseeeeeee justify the significance of small family happy family with any five points .plz help me In a video game, a guppy that escapes a net turns into three goldfish. Each goldfish can turn into two betta fish. Each betta fish can turn into two angelfish. Complete the diagram and write the number of fish at each stage. Write and evaluate an expression for the number of angelfish that can be formed from one guppy help, please This is due in 5min's The monthly cost (in dollars) of water use is a linear function of the amount of water used (in hundreds of cubic feet, HCF). The cost for using 13 HCF of water is$34.58, and the cost for using 24 HCF is $53.83. What is the cost for using 19 HCF of water? Type the correct answer in the box. Spell all words correctly.After the filming is finished, Rachel is responsible for arranging individual shots in a proper sequence. Which role is she playingRachel as playing the role of a(n) Fill in the y-value for each box in the t-chart using the equation below !? mrs patrick is employed by a clothing store.She recieves a 6% commission plus a small sallary. Last week she sold $8419 in clothing how much commission did mrs patrick receive this is a story problem btw :) How many grams of nitric acid, HNO3, are required to neutralize (completely react with) 4.30 g of Ca(OH)2? A box of cereal costs $3.69. The box has 13.7 ounces of cereal. About how much does each ounce cost? 2. T llenars las palabras enOuna buscapalabraslas sendasunasdappasun crucigramaBrainliest!!!!